Solved papers for CET Karnataka Engineering CET - Karnataka Engineering Solved Paper-2010

done CET - Karnataka Engineering Solved Paper-2010 Total Questions - 179

  • question_answer1)  All capacitors used in the diagram are identical and each is of capacitance C. Then the effective capacitance between the point A and B is

    A)
    1.5 C

    B)
    6 C

    C)
    C

    D)
    3 C

    View Answer play_arrow
  • question_answer2)  Two identical conducting balls A and B have positive charges \[{{q}_{1}}\] and \[{{q}_{2}}\] respectively but \[{{q}_{1}}\ne {{q}_{2}}.\] The balls are brought together so that they touch each other and then kept in their original positions. The force between them is

    A)
    less than that before the balls touched

    B)
    greater than that before the balls touched

    C)
    same as that before the balls touched

    D)
    zero

    View Answer play_arrow
  • question_answer3)    Red light of wavelength 625 nm is incident normally on an optical diffraction grating with \[2\times {{10}^{5}}\] lines/m. Including central principal maxima, how many maxima may be observed on a screen which is far from the grating?  

    A)
    15

    B)
    17

    C)
    8

    D)
    16

    View Answer play_arrow
  • question_answer4)  A battery of emf E has an internal resistance r. A variable resistance R is connected to the terminals of the battery. A current; is drawn from the battery. V is the terminal potential difference. If R alone is gradually reduced to zero, which of the following best describes \[i\]and V?

    A)
    \[i\]approaches zero, V approaches E

    B)
    \[i\] approaches \[\frac{E}{r}\], V approaches zero

    C)
    \[i\] approaches \[\frac{E}{r}\], V approaches E

    D)
    \[i\] approaches infinity, V approaches E

    View Answer play_arrow
  • question_answer5)  Three voltmeters A, B and C having resistances R, 1.5R and 3 R respectively are used in a circuit as shown. When a potential difference is applied between X and Y, the readings of the voltmeters are \[{{V}_{1}},\text{ }{{V}_{2}}\], and \[{{V}_{3}}\]respectively. Then

    A)
    \[{{V}_{1}}={{V}_{2}}={{V}_{3}}\]

    B)
    \[{{V}_{1}}<{{V}_{2}}={{V}_{3}}\]

    C)
    \[{{V}_{1}}>{{V}_{2}}>{{V}_{3}}\]

    D)
    \[{{V}_{1}}>{{V}_{2}}={{V}_{3}}\]

    View Answer play_arrow
  • question_answer6)  A point object O is placed in front of a glass rod having spherical end of radius of curvature 30 cm. The image would be formed at

    A)
    30 cm left           

    B)
    infinity

    C)
    1 cm to the right              

    D)
    18 cm to the left

    View Answer play_arrow
  • question_answer7)  A, B and C are the parallel sided transparent media of refractive indices \[{{n}_{1}},{{n}_{2}}\] and \[{{n}_{3}}\]respectively. They are arranged as shown in the figure. A ray is incident at an angle \[i\] on the surface of separation of A and B which is as shown in the figure. After the refraction into the medium B, the ray grazes the surface of separation of the madia B and C. Then, \[\sin i\] equals to

    A)
    \[\frac{{{n}_{3}}}{{{n}_{1}}}\]                                    

    B)
    \[\frac{{{n}_{1}}}{{{n}_{3}}}\]

    C)
    \[\frac{{{n}_{2}}}{{{n}_{3}}}\]                                   

    D)
    \[\frac{{{n}_{1}}}{{{n}_{2}}}\]

    View Answer play_arrow
  • question_answer8)  A boat has green light of wavelength \[\lambda =500\,nm\] on the mast. What wavelength would be measured and what colour would be observed for this light as seen by a diver submerged in water by the side of the boat? Given, \[{{n}_{w}}=\frac{4}{3}\].

    A)
    Green of wavelength 376 nm

    B)
    Red of wavelength 665 nm

    C)
    Green of wavelength 500 nm

    D)
    Blue of wavelength 376 nm

    View Answer play_arrow
  • question_answer9)  Two beams of red and violet colours are made to pass separately through a prism of \[A={{60}^{o}}\]. In the minimum deviation position, the angle of refraction inside the prism will be

    A)
    greater for red colour

    B)
    equal but not \[{{30}^{o}}\] for both the colours

    C)
    greater for violet colour

    D)
    \[{{30}^{o}}\] for both the colours

    View Answer play_arrow
  • question_answer10)  A plano-convex lens is made of refractive index of 1.6. The radius of curvature of the curved surface is 60 cm. The focal length of the lens is

    A)
    400 cm

    B)
      200 cm

    C)
      100 cm                 

    D)
      50 cm

    View Answer play_arrow
  • question_answer11)  Two simple harmonic motions are represented by \[{{y}_{1}}=5[\sin 2\pi t+\sqrt{3}\cos 2\pi t]\]and \[{{y}_{2}}=5\sin \left( 2\pi t+\frac{\pi }{4} \right)\] The ratio of their amplitudes is

    A)
    \[1:1\]                                  

    B)
    \[2:1\]

    C)
    \[1:3\]                                  

    D)
    \[\sqrt{3}:1\]

    View Answer play_arrow
  • question_answer12)  A bat flies at a steady speed of \[4\text{ }m{{s}^{-1}}\] emitting a sound of \[f=90\times {{10}^{3}}Hz\]. It is flying horizontally towards a vertical wall. The frequency of the reflected sound as detected by the bat will be (take velocity of sound in air is \[330\text{ }m{{s}^{-1}}\])

    A)
    \[88.1\times {{10}^{3}}Hz\]        

    B)
    \[87.1\times {{10}^{3}}Hz\]

    C)
    \[92.1\times {{10}^{3}}Hz\]        

    D)
    \[89.1\times {{10}^{3}}Hz\]

    View Answer play_arrow
  • question_answer13)  A closed organ pipe and an open organ pipe of same length produce 2 beats/second while vibrating in their fundamental modes. The length of the open organ pipe is halved and that of closed pipe is doubled. Then the number of beats produced per second while vibrating in the fundamental mode is

    A)
    2

    B)
    6

    C)
    8

    D)
    7

    View Answer play_arrow
  • question_answer14)  A uniform wire of length L, diameter D and density \[\rho \] is stretched under a tension T. The correct relation between, its fundamental frequency \[f\], the length L and the diameter D is

    A)
    \[f\propto \frac{1}{LD}\]                             

    B)
    \[f\propto \frac{1}{L\sqrt{D}}\]

    C)
    \[f\propto \frac{1}{{{D}^{2}}}\]                

    D)
    \[f\propto \frac{1}{L{{D}^{2}}}\]

    View Answer play_arrow
  • question_answer15)  Two small spheres of masses \[{{M}_{1}}\] and \[{{M}_{2}}\]are suspended by weightless insulating threads of lengths \[{{L}_{1}}\] and \[{{L}_{2}}\]. The spheres carry charges \[{{Q}_{1}}\] and \[{{Q}_{2}}\] respectively. The spheres are suspended such that they are in level with one another and the threads are inclined to the vertical at angles of \[{{\theta }_{1}}\] and \[{{\theta }_{2}}\] as shown. Which one of the following conditions is essential, if \[{{\theta }_{1}}={{\theta }_{2}}\]?

    A)
    \[{{M}_{1}}\ne {{M}_{2}},\] but \[{{Q}_{1}}={{Q}_{2}}\]

    B)
    \[{{M}_{1}}={{M}_{2}}\]

    C)
    \[{{Q}_{1}}={{Q}_{2}}\]                

    D)
    \[{{L}_{1}}={{L}_{2}}\]

    View Answer play_arrow
  • question_answer16)  The wavelength of the light used in Young's double slit experiment is \[\lambda \]. The intensity at a point on the screen is \[I\], where the path difference is \[\frac{\lambda }{6}\]. If \[{{I}_{0}}\] denotes the maximum intensity, then the ratio of \[I\] and \[{{I}_{0}}\] is

    A)
    0.866                                    

    B)
    0.5

    C)
    0.707                                    

    D)
    0.75

    View Answer play_arrow
  • question_answer17)  What is the minimum thickness of a thin film required for constructive interference in the reflected light from it? Given, the refractive index of the film = 1.5, wavelength of the light incident on the film = 600 nm.

    A)
    100 nm                                

    B)
    300 nm

    C)
    50 nm                                   

    D)
    200 nm

    View Answer play_arrow
  • question_answer18)  There is a uniform electric field of intensity E which is as shown. How many labelled points have the same electric potential as the fully shaded point?  

    A)
    2

    B)
    3

    C)
    8

    D)
    11

    View Answer play_arrow
  • question_answer19)  Critical angle for certain medium is \[si{{n}^{-1}}(0.6)\]. The polarizing angle of that medium is

    A)
    \[ta{{n}^{-1}}[1.5]\]                       

    B)
    \[{{\sin }^{-1}}[0.8]\]

    C)
    \[{{\tan }^{-1}}[1.6667]\]             

    D)
    \[{{\tan }^{-1}}[0.6667]\]

    View Answer play_arrow
  • question_answer20)  The speed of electromagnetic wave in vacuum depends upon the source of radiation

    A)
    increases as we move from \[\gamma \]-rays to radio waves

    B)
    decreases as we move from \[\gamma \]-rays to radio waves

    C)
    is same for all of them

    D)
    None of the above

    View Answer play_arrow
  • question_answer21)  The moment of inertia of a circular disc of radius 2 m and mass 1 kg about an axis passing through the centre of mass but perpendicular to the plane of the disc is\[2\text{ }kg\text{ }{{m}^{2}}\]. Its moment of inertia about an axis parallel to this axis but passing through the edge of the disc is (see the given figure).

    A)
    \[8\text{ }kg\text{ }{{m}^{2}}\]                

    B)
    \[4\text{ }kg\text{ }{{m}^{2}}\]

    C)
    \[10\text{ }kg\text{ }{{m}^{2}}\]                              

    D)
    \[6\text{ }kg\text{ }{{m}^{2}}\]

    View Answer play_arrow
  • question_answer22)  An astronaut on a strange planet finds that acceleration due to gravity is twice as that on the surface of earth. Which of the following could explain this?

    A)
    Both the mass and radius of the planet are half as that of earth

    B)
    Radius of the planet is half as that of earth, but the mass is the same as that of earth

    C)
    Both the mass and radius of the planet are twice as that of earth

    D)
    Mass of the planet is half as that of earth, but radius is same as that of earth

    View Answer play_arrow
  • question_answer23)  Which of the following substances has the highest elasticity?

    A)
    Sponge                                

    B)
    Steel

    C)
    Rubber                                

    D)
    Copper

    View Answer play_arrow
  • question_answer24)  Three liquids of equal masses are taken in three identical cubical vessels A, B and C. Their densities are \[{{\rho }_{A}},{{\rho }_{B}}\] and \[{{\rho }_{C}}\]respectively but \[{{\rho }_{A}}<{{\rho }_{B}}<{{\rho }_{C}}.\] The force exerted by the liquid on the base of the cubical vessel is

    A)
    maximum in vessel C

    B)
    minimum in vessel C

    C)
    the same in all the vessels

    D)
    maximum in vessel A

    View Answer play_arrow
  • question_answer25)  Water is in streamline flows along a horizontal pipe with non-uniform cross-section. At a point in the pipe where the area of cross section is \[10\text{ }c{{m}^{2}}\], the velocity of water is \[1\text{ }m{{s}^{-1}}\] and the pressure is 2000 Pa. The pressure at another point where the cross sectional area is \[5\text{ }c{{m}^{2}}\] is

    A)
    4000 Pa                

    B)
    2000 Pa

    C)
    1000 Pa                

    D)
    500 Pa

    View Answer play_arrow
  • question_answer26)  In the circuit given here, the points A, B and C are 70 V, zero, 10 V respectively. Then

    A)
    the point D will be at a potential of 60 V

    B)
    the point D will be at a potential of 20 V

    C)
    currents in the paths AD, DB and DC are in the ratio of \[1:2:3\]

    D)
     currents in the paths AB, DB and DC are in the ratio of \[3:2:1\]

    View Answer play_arrow
  • question_answer27)  \[{{B}_{1}},{{B}_{2}}\] and \[{{B}_{3}}\] are the three identical bulbs connected to a battery of steady emf with key K closed. What happens to the brightness of the bulbs \[{{B}_{1}}\] and \[{{B}_{2}}\] when the key is opened?

    A)
    Brightness of the bulb \[{{B}_{1}}\] increases and that of \[{{B}_{2}}\] decreases

    B)
    Brightness of the bulbs \[{{B}_{1}}\] and \[{{B}_{2}}\]increase

    C)
    Brightness of the bulb \[{{B}_{1}}\] decreases and \[{{B}_{2}}\]increases

    D)
    Brightness of the bulbs \[{{B}_{1}}\] and \[{{B}_{2}}\]decrease

    View Answer play_arrow
  • question_answer28)  Magnetic field at the centre of a circular coil of radius R due to \[i\] flowing through it is B. The magnetic field at a point along the axis at distance R from the centre is

    A)
    \[\frac{B}{2}\]                                  

    B)
    \[\frac{B}{4}\]

    C)
    \[\frac{B}{\sqrt{8}}\]                     

    D)
    \[\sqrt{8}B\]

    View Answer play_arrow
  • question_answer29)  Two thick wires and two thin wires, all of same material and same length, form a square in three different ways P, Q and R as shown in the figure. With correct connections shown, the magnetic field due to the current flow, at the centre of the loop will be zero in case of

    A)
    Q and R                               

    B)
    P only

    C)
    P and Q               

    D)
    P and R

    View Answer play_arrow
  • question_answer30)  There is a uniform magnetic field directed perpendicular and into the plane of the paper. An irregular shaped conducting loop is slowly changing into a circular loop in the plane of the paper. Then

    A)
    current is induced in the loop in the anticlockwise direction

    B)
    current is induced in the loop in the clockwise direction

    C)
    AC is induced in the loop

    D)
    no current is induced in the loop

    View Answer play_arrow
  • question_answer31)  The dimensions of resistance are same as those of..... .where h is the Planck's constant, e is the charge.

    A)
    \[\frac{{{h}^{2}}}{{{e}^{2}}}\]                                    

    B)
    \[\frac{{{h}^{2}}}{e}\]

    C)
    \[\frac{h}{{{e}^{2}}}\]                                   

    D)
    \[\frac{h}{e}\]

    View Answer play_arrow
  • question_answer32)  A train is moving slowly on a straight track with a constant speed of \[2\text{ }m{{s}^{-1}}\]. A passenger in that train starts walking at a steady speed of \[2\text{ }m{{s}^{-1}}\] to the back of the train in the opposite direction of the motion of the train. So to an observer standing on the platform directly in front of that passenger, the velocity of the passenger appears to be

    A)
    \[4\,m{{s}^{-1}}\]

    B)
    \[2\,m{{s}^{-1}}\]

    C)
    \[2\,m{{s}^{-1}}\] in the opposite direction of the train

    D)
    zero

    View Answer play_arrow
  • question_answer33)  A ball rests upon a flat piece of paper on a table top. The paper is pulled horizontally but quickly towards right as shown relative to its initial position with respect to the table, the ball

    A)
    Both (1) and (2)               

    B)
    only (3)

    C)
    only (1)                

    D)
    only (2)                

    View Answer play_arrow
  • question_answer34)  A boy throws a cricket ball from the boundary to the wicket-keeper. If the frictional force due to air cannot be ignored, the forces acting on the ball at the position X are respected by

    A)

    B)

    C)

    D)

    View Answer play_arrow
  • question_answer35)  If the linear momentum of a body is increased by 50%, then the kinetic energy of that body increases by

    A)
    100%                                    

    B)
    125%

    C)
    225%                                    

    D)
    25%

    View Answer play_arrow
  • question_answer36)  The temperature of a gas contained in a closed vessel of constant volume increases by \[{{1}^{o}}C\] when the pressure of the gas is increased by 1%. The initial temperature of the gas is

    A)
    100 K                                    

    B)
    \[{{273}^{o}}C\]

    C)
    \[{{100}^{o}}C\]               

    D)
    200 K

    View Answer play_arrow
  • question_answer37)  A motorboat covers a given distance in 6 h moving downstream on a river. It covers the same distance in 10 h moving upstream. The time it takes to cover the same distance in still water is

    A)
    9 h                                         

    B)
    7.5 h

    C)
    6.5 h                                     

    D)
    8 h

    View Answer play_arrow
  • question_answer38)  Two slabs are of the thicknesses \[{{d}_{1}}\] and \[{{d}_{2}}\]. Their thermal conductivities are \[{{K}_{1}}\] and \[{{K}_{2}}\] respectively. They are in series. The free ends of the combination of these two slabs are kept at temperatures \[{{\theta }_{1}}\] and \[{{\theta }_{2}}\]. Assume \[{{\theta }_{1}}>{{\theta }_{2}}\]. The temperature \[\theta \] of their common junction is

    A)
      \[\frac{{{K}_{1}}{{\theta }_{1}}+{{K}_{2}}{{\theta }_{2}}}{{{\theta }_{1}}+{{\theta }_{2}}}\]                           

    B)
      \[\frac{{{K}_{1}}{{\theta }_{1}}{{d}_{1}}+{{K}_{2}}{{\theta }_{2}}{{d}_{2}}}{{{K}_{1}}{{d}_{2}}+{{K}_{2}}{{d}_{1}}}\]

    C)
      \[\frac{{{K}_{1}}{{\theta }_{1}}{{d}_{2}}+{{K}_{2}}{{\theta }_{2}}{{d}_{1}}}{{{K}_{1}}{{d}_{2}}+{{K}_{2}}{{d}_{1}}}\]               

    D)
      \[\frac{{{K}_{1}}{{\theta }_{1}}+{{K}_{2}}{{\theta }_{2}}}{{{K}_{1}}+{{K}_{2}}}\]

    View Answer play_arrow
  • question_answer39)  Hot water cools from \[{{60}^{o}}C\] to \[{{50}^{o}}C\] in the first 10 min and to \[{{42}^{o}}C\] in the next 10 min. Then the temperature of the surroundings is

    A)
    \[{{20}^{o}}C\]                 

    B)
    \[{{30}^{o}}C\]

    C)
    \[{{15}^{o}}C\]                                 

    D)
    \[{{10}^{o}}C\]

    View Answer play_arrow
  • question_answer40)  The efficiency of Carnot's heat heat engine is 0.5 when the temperature of the source is \[{{T}_{1}}\]that of sink is \[{{T}_{2}}\]. The efficiency of source. Carnot's heat engine is also 0.5. The temperatures of source and second engine are respectively

    A)
    \[2{{T}_{1}},2{{T}_{2}}\]                               

    B)
    \[2{{T}_{1}},\frac{{{T}_{2}}}{2}\]

    C)
    \[{{T}_{1}}+5,\,{{T}_{2}}-5\]       

    D)
    \[{{T}_{1}}+10,\,{{T}_{2}}-10\]  

    View Answer play_arrow
  • question_answer41)  A current \[i\] is flowing through the loop. The direction of the current and the shape of the loop are as shown in the figure. The magnetic field at the centre of the loop is \[\frac{{{\mu }_{0}}i}{R}\] times

    A)
      \[\frac{5}{16},\] but out of the plane of the paper

    B)
      \[\frac{5}{16},\] but into the plane of the paper

    C)
      \[\frac{7}{16},\] but out of the plane of the paper

    D)
      \[\frac{7}{16},\] but into the plane of the paper

    View Answer play_arrow
  • question_answer42)  An ideal choke draws a current of 8 A when connected to an AC supply of 100 V, 50 Hz. A pure resistor draws a current of 10 A when connected to the same source. The ideal choke and the resistor are connected in series and then connected to the AC source of 150 V, 40 Hz. The current in the circuit becomes

    A)
      \[\frac{15}{\sqrt{2}}A\]                                

    B)
      8 A

    C)
      18 A                                      

    D)
      10 A

    View Answer play_arrow
  • question_answer43)  The spectrum of an oil flame is an example for

    A)
    line emission spectrum

    B)
    continuous emission spectrum

    C)
    line absorption spectrum

    D)
    band emission spectrum

    View Answer play_arrow
  • question_answer44)  According to Einstein's photoelectric equation, the graph of KE of the photoelectron emitted from the metal versus the frequency of the incident radiation gives a straight line graph, whose slope

    A)
    depends on the intensity of the incident radiation

    B)
    depends on the nature of the metal and also on the intensity of incident radiation

    C)
    is same for all metals and independent of the intensity of the incident radiation

    D)
    depends on the nature of the metal

    View Answer play_arrow
  • question_answer45)  An electron is moving in an orbit of a hydrogen atom from which there can be a maximum of six transition. An electron is moving in an orbit of another hydrogen atom from which there can be a maximum of three transition. The ratio of the velocities of the electron in these two orbits is

    A)
    \[\frac{1}{2}\]                                   

    B)
    \[\frac{2}{1}\]

    C)
    \[\frac{5}{4}\]                                   

    D)
    \[\frac{3}{4}\]

    View Answer play_arrow
  • question_answer46)  \[{{v}_{1}}\] is the frequency of the series limit of Lyman series, \[{{v}_{2}}\] is the frequency of the first line of Lyman series and \[{{v}_{3}}\] is the frequency of the series limit of the Balmer series. Then

    A)
    \[{{v}_{1}}-{{v}_{2}}={{v}_{3}}\]                               

    B)
    \[{{v}_{1}}={{v}_{2}}-{{v}_{3}}\]

    C)
    \[\frac{1}{{{v}_{1}}}=\frac{1}{{{v}_{1}}}+\frac{1}{{{v}_{3}}}\]      

    D)
    \[\frac{1}{{{v}_{1}}}=\frac{1}{{{v}_{1}}}+\frac{1}{{{v}_{3}}}\]

    View Answer play_arrow
  • question_answer47)  Assume the graph of specific binding energy versus mass number is as shown in the figure. Using this graph, select the correct choice from the following.

    A)
      Fusion of two nuclei of mass number lying in the range of \[100<A<200\] will release energy

    B)
      Fusion of two nuclei of mass number lying in the range of \[51<A<100\] will release energy

    C)
      Fusion of two nuclei of mass number lying in the range of \[1<A<50\] will release energy

    D)
    Fission of the nucleus of mass number lying in the rang of \[100<A<200\] will release energy when broken into two fragments

    View Answer play_arrow
  • question_answer48)  Pick out the correct statement from the following.

    A)
    Energy released per unit mass of the reactant is less in case of fusion reaction

    B)
    Packing fraction may be positive or may be negative

    C)
    \[P{{u}^{239}}\] is not suitable for a fission reaction

    D)
    For stable nucleus, the specific binding energy is low

    View Answer play_arrow
  • question_answer49)  A radioactive sample \[{{S}_{1}}\] having the activity \[{{A}_{1}}\] has twice the number of nuclei as another sample \[{{S}_{2}}\] of activity \[{{A}_{2}}\]. If \[{{A}_{2}}=2{{A}_{1}}\], then the ratio of half-life of \[{{S}_{1}}\] to the half-life of \[{{S}_{2}}\] is

    A)
    4

    B)
    2

    C)
    0.25

    D)
    0.75

    View Answer play_arrow
  • question_answer50)  When a neutron is disintegrated to give a \[\beta \]-particle,

    A)
    a neutrino alone is emitted

    B)
    a proton and neutrino are emitted

    C)
    a proton alone is emitted

    D)
    a proton and an antineutrino are emitted

    View Answer play_arrow
  • question_answer51)  The forbidden energy gap in Ge is 0.72 eV, Given, \[hc=12400\text{ }eV-\overset{o}{\mathop{A}}\,\]. The maximum wavelength of radiation that will generate electron hole pair is

    A)
    \[172220\,\,\overset{\text{o}}{\mathop{\text{A}}}\,\]

    B)
      \[172.2\,\,\overset{\text{o}}{\mathop{\text{A}}}\,\]  

    C)
    \[17222\,\,\overset{\text{o}}{\mathop{\text{A}}}\,\]

    D)
    \[1722\,\,\overset{\text{o}}{\mathop{\text{A}}}\,\]

    View Answer play_arrow
  • question_answer52)  In a p-n junction diode not connected to any circuit

    A)
    the potential is the same everywhere

    B)
    the p-type side has a higher potential than the n-type side

    C)
    there is an electric field at the junction  directed from the n-type side to p-type side

    D)
    there is an electric field at the junction directed from the p-type side to n-type side

    View Answer play_arrow
  • question_answer53)  In a given direction, the intensities of the scattered light by a scattering substance for two beams of light are in the ratio of \[256:81\]. The ratio of the frequency of the first beam to the frequency of the second beam is

    A)
    \[64:127\]                           

    B)
    \[1:2\]

    C)
    \[64:27\]                             

    D)
    None of these

    View Answer play_arrow
  • question_answer54)  Identify the logic operation performed by the circuit given here.

    A)
    OR                                         

    B)
    NOR

    C)
    NOT                                      

    D)
    NAND

    View Answer play_arrow
  • question_answer55)  The de-Broglie wavelength of the electron in the ground state of the hydrogen atom is ...... (radius of the first orbit of hydrogen atom\[=0.53\overset{o}{\mathop{A}}\,\]).

    A)
    \[1.67\overset{o}{\mathop{A}}\,\]                         

    B)
    \[3.33\overset{o}{\mathop{A}}\,\]

    C)
    \[1.06\overset{o}{\mathop{A}}\,\]                         

    D)
    \[0.53\overset{o}{\mathop{A}}\,\]

    View Answer play_arrow
  • question_answer56)  PQ and RS are long parallel conductors   separated by certain distance. M is the midpoint between them (see the figure). The net magnetic field at M is B. Now, the current 2 A is switched off. The field at M now becomes

    A)
    2 B                                         

    B)
    B

    C)
    \[\frac{B}{2}\]

    D)
    35

    View Answer play_arrow
  • question_answer57)  An electron enters the space between the plates of a charged capacitor as shown. The charge density on the plate is \[\sigma \]. Electric intensity in the space between the plates is A uniform magnetic field B also exists in the space perpendicular to the direction of E. The electron moves perpendicular to both \[\overrightarrow{E}\] and \[\overrightarrow{B}\] without any change in direction. The time taken by the electron to travel a distance l in the space is

    A)
    \[\frac{\sigma l}{{{\varepsilon }_{0}}B}\]                              

    B)
    \[\frac{\sigma B}{{{\varepsilon }_{0}}l}\]

    C)
    \[\frac{{{\varepsilon }_{0}}lB}{\sigma B}\]                           

    D)
    \[\frac{{{\varepsilon }_{0}}l}{\sigma B}\]

    View Answer play_arrow
  • question_answer58)  In a series resonant R-L-C circuit, the voltage across R is 100 V and the value of \[R=1000\,\Omega \]. The capacitance of the capacitor is \[2\times {{10}^{-6}}F\]; angular frequency of AC is \[200\text{ }rad\text{ }{{s}^{-1}}\]. Then the potential difference across the inductance coil is

    A)
    100 V                                    

    B)
    40 V

    C)
    250 V                                    

    D)
    400 V

    View Answer play_arrow
  • question_answer59)  A capacitor and an inductance coil are connected in separate AC circuits with a bulb glowing in both the circuits. The bulb glows more brightly when

    A)
    an iron rod is introduced into the inductance coil

    B)
    the number of turns in the inductance coil is increased

    C)
    separation between the plates of the capacitor is increased

    D)
    a dielectric is introduced into the gap between the plates of the capacitor

    View Answer play_arrow
  • question_answer60)  A charge +Q is moving upwards vertically. It enters a magnetic field directed to north. The force on the charge will be towards

    A)
    north                                    

    B)
    south

    C)
    east                                      

    D)
    west

    View Answer play_arrow
  • question_answer61)  In the electrolytic refining of zinc

    A)
    graphite is at the anode

    B)
    the impure metal is at the cathode

    C)
    the metal ion get reduced at the anode

    D)
    acidified zinc sulphate is the electrolyte

    View Answer play_arrow
  • question_answer62)  The wave number of the spectral line in the emission spectrum of hydrogen will be equal to \[\frac{8}{9}\] times the Rydberg's constant if the electron jumps from

    A)
    \[n=3\] to \[n=1\]           

    B)
    \[n=10\] to \[n=1\]

    C)
    \[n=9\] to \[n=1\]           

    D)
    \[n=2\] to \[n=1\]

    View Answer play_arrow
  • question_answer63)  Consider the following gaseous equilibria with equilibrium constants \[{{K}_{1}}\]and \[{{K}_{2}}\] respectively. \[S{{O}_{2}}(g)+\frac{1}{2}{{O}_{2}}(g)S{{O}_{3}}(g)\] \[2S{{O}_{3}}(g)2S{{O}_{2}}(g)+{{O}_{2}}(g)\] The equilibrium constants are related as

    A)
    \[K_{1}^{2}=\frac{1}{{{K}_{2}}}\]             

    B)
    \[2{{K}_{1}}=K_{2}^{2}\]

    C)
    \[{{K}_{2}}=\frac{2}{K_{1}^{2}}\]                             

    D)
    \[K_{2}^{2}=\frac{1}{{{K}_{1}}}\]

    View Answer play_arrow
  • question_answer64)  Enthalpy of vaporization of benzene is \[+\,\,35.3\,kJ\,\,mo{{l}^{-1}}\] at its boiling point, \[{{80}^{o}}C\]. The entropy change in the transition of the vapour to liquid at its boiling point in \[[J{{K}^{-1}}mo{{l}^{-1}}]\] is

    A)
    -441                                      

    B)
    -100

    C)
    +441                                     

    D)
    +100

    View Answer play_arrow
  • question_answer65)  Which one of the following conversions involve change in both hybridisation and shape?

    A)
      \[C{{H}_{4}}\xrightarrow{{}}{{C}_{2}}{{H}_{6}}\]               

    B)
      \[N{{H}_{3}}\xrightarrow{{}}NH_{4}^{+}\]

    C)
      \[B{{F}_{3}}\xrightarrow{{}}BF_{4}^{-}\]              

    D)
      \[{{H}_{2}}O\xrightarrow{{}}{{H}_{3}}{{O}^{+}}\]

    View Answer play_arrow
  • question_answer66)  In chromite ore, the oxidation number of iron, and chromium are respectively

    A)
      + 3, + 2                                   

    B)
    + 3, + 6

    C)
    + 2, + 6                                 

    D)
    + 2, + 3

    View Answer play_arrow
  • question_answer67)  For the reversible reaction. \[A\,(s)+B(g)C\,(g)+D(g);\]\[\Delta {{G}^{o}}=-350\,\,kJ\] Which one of the following statements is true?

    A)
    The entropy change is negative

    B)
    Equilibrium constant is greater than one

    C)
    The reaction should be instantaneous

    D)
    The reaction is thermodynamically not feasible

    View Answer play_arrow
  • question_answer68)  Out of the compounds below the vapour pressure of at a particular temperature is

    A)
    higher than that of

    B)
    lower than that of

    C)
    higher or lower than, depending on the size of the vessel

    D)
    same as that of 

    View Answer play_arrow
  • question_answer69)  The amount of heat evolved when \[500\text{ }c{{m}^{3}}\] of \[0.1\text{ }M\,HCl\] is mixed with \[200\text{ }c{{m}^{3}}\] of \[0.2\text{ }NaOH\] is

    A)
    2.292 kJ               

    B)
    1.292kJ

    C)
    0.292 kJ                               

    D)
    3.392 kJ

    View Answer play_arrow
  • question_answer70)  During the adsorption of krypton on activated charcoal at low temperature

    A)
    \[\Delta H>0\] and \[\Delta S<0\]

    B)
    \[\Delta H<0\] and \[\Delta S<0\]

    C)
    \[\Delta H>0\] and \[\Delta S>0\]

    D)
    \[\Delta H<0\] and \[\Delta S>0\]

    View Answer play_arrow
  • question_answer71)  The set of quantum numbers for the outermost electron for copper in its ground state is

    A)
    \[4,\,\,1,\,\,1,+\frac{1}{2}\]        

    B)
    \[3,\,\,2,\,\,2,+\frac{1}{2}\]

    C)
    \[4,\,\,0,\,\,0,+\frac{1}{2}\]        

    D)
    \[4,\,\,2,\,\,2,+\frac{1}{2}\]

    View Answer play_arrow
  • question_answer72)  Peroxide ion ......

    A)
    (ii) and (iii)          

    B)
    (i), (ii) and (iv)

    C)
    (i), (ii) and (iii)  

    D)
    (i) and (iv)

    View Answer play_arrow
  • question_answer73)  Which one of these is not true for benzene?

    A)
    It forms only one type of mono substituted product

    B)
    There are three carbon-carbon single bonds and three carbon-carbon double bonds

    C)
    The heat of hydrogenation of benzene is less than the theoretical value

    D)
    The bond angle between the carbon-carbon bonds is \[{{120}^{o}}\]

    View Answer play_arrow
  • question_answer74)  A mixture of \[CaC{{l}_{2}}\] and \[NaCl\] weighing 4.44 g is treated with sodium carbonate solution to precipitate all the \[C{{a}^{2+}}\] ions as calcium carbonate. The calcium carbonate so obtained is heated strongly to get 0.56 g of CaO. The percentage of \[NaCl\] in the mixture (atomic mass of \[Ca=40\]) is

    A)
    75                                          

    B)
    30.6

    C)
    25                                          

    D)
    69.4

    View Answer play_arrow
  • question_answer75)  For one mole of an ideal gas, increasing the temperature from \[{{10}^{o}}C\] to \[{{20}^{o}}C\]

    A)
    increases the average kinetic energy by two times

    B)
    increases the rms velocity by \[\sqrt{2}\] times

    C)
    increases the rms velocity by two times

    D)
    increases both the average kinetic energy and rms velocity, but not significantly

    View Answer play_arrow
  • question_answer76)  Generally, the first ionization energy increases along a period. But there are some exceptions. One which is not an exception is

    A)
    N and O               

    B)
    Na and Mg

    C)
    Mg and Al           

    D)
    Be and B

    View Answer play_arrow
  • question_answer77)  \[50\,\,c{{m}^{3}}\] of \[0.2\text{ }N\,HCl\] is titrated against \[0.1\text{ }N\,NaOH\] solution. The titration is discontinued after adding \[50\,\,c{{m}^{3}}\] of \[NaOH\]. The remaining titration is completed by adding \[0.5\text{ }N\,KOH\]. The volume of \[KOH\]required for completing the titration is

    A)
    \[12\,\,c{{m}^{3}}\]                       

    B)
    \[10\,\,c{{m}^{3}}\]

    C)
    \[25\,\,c{{m}^{3}}\]                       

    D)
    \[10.5\,\,c{{m}^{3}}\]

    View Answer play_arrow
  • question_answer78)  In which one of the following, does the given amount of chlorine exert the least pressure in a vessel of capacity \[1\,\,d{{m}^{3}}\] at 273 K?

    A)
    0.0355 g

    B)
    0.071 g

    C)
    \[6.023\times {{20}^{21}}\]molecules

    D)
    0.02 mol

    View Answer play_arrow
  • question_answer79)  Based on the first law of thermodynamics, which one of the following is correct?

    A)
    For an isochoric process \[=\Delta E=-q\]

    B)
    For an adiabatic process \[=\Delta E=-w\]

    C)
    For an isothermal process \[=q=+w\]

    D)
    For a cyclic process \[q=-w\]

    View Answer play_arrow
  • question_answer80)  For alkali metals, which one of the following trends is incorrect?

    A)
    Hydration energy: \[Li>Na>K>Rb\]

    B)
    Ionization energy: \[Li>Na>K>Rb\]

    C)
    Density: \[Li<Na<K<Rb\]

    D)
    Atomic size: \[Li<Na<K<Rb\]

    View Answer play_arrow
  • question_answer81)  1 g of silver gets distributed between \[10\,\,c{{m}^{3}}\]of molten zinc and \[100\,\,c{{m}^{3}}\] of molten lead at\[{{800}^{o}}C\]. The percentage of silver in the zinc layer is approximately

    A)
    89

    B)
    91

    C)
    97

    D)
    94

    View Answer play_arrow
  • question_answer82)  One mole of an organic compound A with the formula \[{{C}_{3}}{{H}_{8}}O\] reacts completely with two moles of HI to form X and Y. When Y is boiled with aqueous alkali it forms Z. Z answers the iodoform test. The compound A is

    A)
    propan-2-ol       

    B)
    propan-1-ol

    C)
    ethoxyethane                  

    D)
    methoxyethane

    View Answer play_arrow
  • question_answer83)    The IUPAC name of \[{{K}_{2}}[Ni{{(CN)}_{4}}]\] is  

    A)
    potassium tetracyanonickelate (II)

    B)
    potassium tetracyanatonickelate (III)

    C)
    potassium tetracyanatonickelate (II)

    D)
    potassium tetracyanonickelate (III)

    View Answer play_arrow
  • question_answer84)  The spin only magnetic moment of \[M{{n}^{4+}}\] ion is nearly

    A)
    3 BM                                     

    B)
    6 BM

    C)
    4 BM                                     

    D)
    5 BM

    View Answer play_arrow
  • question_answer85)  In Kjeldahl’s method, ammonia from 5 g of food neutralizes \[30\,c{{m}^{3}}\] of 0.1 N acid. The percentage of nitrogen in the food is

    A)
    8.4

    B)
      8.4

    C)
      16.8                                       

    D)
      1.68

    View Answer play_arrow
  • question_answer86)  Carbon can reduce ferric oxide to iron at a temperature above 983 K because

    A)
    carbon monoxide formed is thermodynamically less stable than ferric oxide

    B)
    carbon has a higher affinity towards oxidation than iron

    C)
    free energy change for the formation of carbon dioxide is less negative than that for ferric oxide

    D)
    iron has a higher affinity towards oxygen than carbon

    View Answer play_arrow
  • question_answer87)  An oxygen containing organic compound upon oxidation forms a carboxylic acid as the only organic product with its molecular mass higher by 14 units. The organic compound is

    A)
    an aldehyde

    B)
    a primary alcohol

    C)
    a secondary alcohol

    D)
    acetone

    View Answer play_arrow
  • question_answer88)  The compound obtained when acetaldehyde reacts with dilute aqueous sodium hydroxide exhibits

    A)
    geometrical isomerism

    B)
    optical isomerism

    C)
    neither optical nor geometrical isomerism

    D)
    both optical and geometrical isomerism

    View Answer play_arrow
  • question_answer89)  The activation energy for a reaction at the temperature TK was found to be\[2.303\text{ }RT\text{ }J\text{ }mo{{l}^{-1}}\]. The ratio of the rate constant to Arrhenius factor is

    A)
    \[{{10}^{-1}}\]                                  

    B)
    \[{{10}^{-2}}\]

    C)
    \[2\times {{10}^{-3}}\]                  

    D)
    \[2\times {{10}^{-2}}\]

    View Answer play_arrow
  • question_answer90)  A dibromo derivative of an alkane reacts with sodium metal to form an alicyclic hydrocarbon. The derivative is

    A)
    1,1-dibromopropane

    B)
    2, 2-dibromobutane

    C)
    1, 2-dibromoethane

    D)
    1, 4-dibromobutane

    View Answer play_arrow
  • question_answer91)  Time required for 100 per cent completion of a zero order reaction is

    A)
    \[\frac{2k}{a}\]                                

    B)
    \[\frac{a}{2k}\]

    C)
    \[\frac{a}{k}\]                                   

    D)
    \[ak\]

    View Answer play_arrow
  • question_answer92)  0.023 g of sodium metal is reacted with \[100\text{ }c{{m}^{3}}\] of water. The pH of the resulting solution is

    A)
    10

    B)
    11

    C)
    9

    D)
    12

    View Answer play_arrow
  • question_answer93)  Which one of the following is wrongly matched?

    A)
    \[{{[Cu{{(N{{H}_{3}})}_{4}}]}^{2+}}\]       - Square planar

    B)
    \[[Ni{{(CO)}_{4}}]\]                         - Neutral ligand

    C)
    \[[Fe{{(C{{N}_{6}})}^{3-}}]\]                       - \[s{{p}^{3}}{{d}^{2}}\]

    D)
    \[{{[Co{{(en)}_{3}}]}^{3+}}\]      - Follows EAN rule

    View Answer play_arrow
  • question_answer94)  Which one of the following conformations of cyclohexane is the least stable?

    A)
    Half-chair            

    B)
    Boat

    C)
    Twisted-boat                    

    D)
    Chair

    View Answer play_arrow
  • question_answer95)  Which one of the following is a molecular crystal?

    A)
    Rock salt             

    B)
    Quartz

    C)
    Dry ice                                 

    D)
    Diamond

    View Answer play_arrow
  • question_answer96)  A buffer solution contains 0.1 mole of sodium acetate in \[1000\,\,c{{m}^{3}}\] of 0.1 M acetic acid. To the above buffer solution, 0.1 mole of sodium acetate is further added and dissolved. The pH of the resulting buffer is equal to

    A)
    \[p{{K}_{a}}-\log \,2\]                   

    B)
    \[p{{K}_{a}}\]

    C)
    \[p{{K}_{a}}+\,2\]                           

    D)
    \[p{{K}_{a}}+\log \,2\]

    View Answer play_arrow
  • question_answer97)  Which one of the following has the most nucleophilic nitrogen?

    A)

    B)

    C)

    D)

    View Answer play_arrow
  • question_answer98)  Chloroacetic acid is a stronger acid than acetic acid. This can be explained using

    A)
    -M effect            

    B)
    -I effect

    C)
    + M effect          

    D)
    +I effect

    View Answer play_arrow
  • question_answer99)  The correct sequence of reactions to convert p-nitrophenol into quinol involves

    A)
    reduction, diazotization and hydrolysis

    B)
    hydrolysis, diazotization and reduction

    C)
    hydrolysis, reduction and diazotization

    D)
    diazotization, reduction and hydrolysis

    View Answer play_arrow
  • question_answer100)  \[C{{H}_{3}}C{{H}_{2}}Br\xrightarrow[\Delta ]{aq.\,KOH}a=\xrightarrow[\Delta ]{KMn{{O}_{4}}/{{H}^{+}}}\]\[B\xrightarrow[\Delta ]{N{{H}_{3}}}C\xrightarrow[alkali]{B{{r}_{2}}}D,'D'\] is

    A)
    \[C{{H}_{3}}Br\]                               

    B)
    \[C{{H}_{3}}COON{{H}_{2}}\]

    C)
    \[C{{H}_{3}}N{{H}_{2}}\]                              

    D)
    \[CHB{{r}_{3}}\]

    View Answer play_arrow
  • question_answer101)  The letter ‘D’ in D-glucose signifies

    A)
    configuration at all chiral carbons

    B)
    dextrorotatory

    C)
    that it is a monosaccharide

    D)
    configuration at a particular chiral carbon

    View Answer play_arrow
  • question_answer102)  Reaction of methyl bromide with aqueous sodium hydroxide involves

    A)
    racemization

    B)
    \[{{S}_{N}}1\] mechanism

    C)
    retention of configuration

    D)
    \[{{S}_{N}}2\] mechanism

    View Answer play_arrow
  • question_answer103)  9.65 C of electric current is passed through fused anhydrous magnesium chloride. The magnesium metal thus, obtained is completely converted into a Grignard reagent. The number of moles of the Grignard reagent obtained is

    A)
    \[5\times {{10}^{-4}}\]                  

    B)
    \[1\times {{10}^{-4}}\]

    C)
    \[5\times {{10}^{-5}}\]                                  

    D)
    \[1\times {{10}^{-5}}\]

    View Answer play_arrow
  • question_answer104)  Which one of the following does not involve coagulation?

    A)
    Formation of delta regions

    B)
    Peptization

    C)
    Treatment of drinking water by potash alum

    D)
    Clotting of blood by the use of ferric chloride

    View Answer play_arrow
  • question_answer105)    In alkaline medium, alanine exists predominantly as  

    A)
      anion                                    

    B)
        Zwitterion  

    C)
      cation                                   

    D)
      covalent form

    View Answer play_arrow
  • question_answer106)  The standard emf of galvanic cell involving 3 moles of electrons in its redox reaction is 0.59 V. The equilibrium constant for the reaction of the cell is

    A)
      \[{{10}^{25}}\]                                  

    B)
      \[{{10}^{20}}\]

    C)
      \[{{10}^{15}}\]                                  

    D)
      \[{{10}^{30}}\]

    View Answer play_arrow
  • question_answer107)    Benzaldehyde and acetone can be best distinguished using  

    A)
      Fehling's solution

    B)
      Sodium hydroxide solution

    C)
      2, 4-DNP

    D)
      Tollen's reagent

    View Answer play_arrow
  • question_answer108)    Which one of the following statements is true?  

    A)
      Saponification of oil yields a diol

    B)
      Drying of oil involves hydrolysis

    C)
      Addition of antioxidant to oil minimizes rancidity

    D)
      Refining of oil involves hydrogenation

    View Answer play_arrow
  • question_answer109)    The following data is obtained during the first order thermal decomposition of \[2A(g)\xrightarrow{{}}B(g)+C(s)\]at constant volume and temperature.  
    S.N. Time Total pressure in Pascal
    1. At the end of 10 min 300
    2. After completion 200
      The rate constant in \[{{\min }^{-1}}\] is  

    A)
      0.0693                                  

    B)
      6.93

    C)
      0.00693                

    D)
        69.3  

    View Answer play_arrow
  • question_answer110)  Phenol \[\xrightarrow{X}\] forms a tribromo derivative. ‘’X’’ is

    A)
    bromine in benzene

    B)
    bromine in water

    C)
    potassium bromide solution

    D)
    bromine in carbon tetrachloride at \[{{0}^{o}}C\]

    View Answer play_arrow
  • question_answer111)  The correct sequence of steps involved in the mechanism of Cannizaro’s reaction is

    A)
    nucleophilic attack, transfer of \[{{H}^{-}}\]and transfer of \[{{H}^{+}}\]

    B)
    transfer of \[{{H}^{-}}\], transfer of \[{{H}^{+}}\] and nucleophilic attack

    C)
    transfer if \[{{H}^{+}}\] nucleophilic attack and transfer of \[{{H}^{-}}\]

    D)
    electrophilic attack by \[O{{H}^{-}}\], transfer of\[{{H}^{+}}\] and transfer of \[{{H}^{-}}\]

    View Answer play_arrow
  • question_answer112)  Which one of the following is an example for homogeneous catalysis?

    A)
    Manufacture of sulphuric acid by Contact process

    B)
    Manufacture of ammonia by Habeas process

    C)
    Hydrolysis of sucrose in presence of dilute hydrochloric acid

    D)
    Hydrogenation of oil

    View Answer play_arrow
  • question_answer113)  The empirical formula of a non-electrolyte is \[C{{H}_{2}}O\]. A solution containing 6 g of the compound exerts the same osmotic pressure as that of 0.05 M glucose solution at the same temperature. The molecular formula of the compound is

    A)
      \[{{C}_{2}}{{H}_{4}}{{O}_{2}}\]                  

    B)
      \[{{C}_{3}}{{H}_{6}}{{O}_{3}}\]

    C)
      \[{{C}_{5}}{{H}_{10}}{{O}_{5}}\]               

    D)
      \[{{C}_{4}}{{H}_{8}}{{O}_{4}}\]

    View Answer play_arrow
  • question_answer114)    A white crystalline salt A reacts with dilute \[HCl\] to liberate a suffocating gas B and also forms a yellow precipitate. The gas B turns potassium dichromate acidified with dilute \[{{H}_{2}}S{{O}_{4}}\] to a green coloured solution C. A, B and C are respectively  

    A)
    \[N{{a}_{2}}S{{O}_{3}},\,S{{O}_{2}}\,\,C{{r}_{2}}{{(S{{O}_{4}})}_{3}}\]

    B)
    \[N{{a}_{2}}{{S}_{2}}{{O}_{3}},\,S{{O}_{2}}\,\,C{{r}_{2}}{{(S{{O}_{4}})}_{3}}\]

    C)
    \[N{{a}_{2}}S,\,\,S{{O}_{2}},\,\,\,C{{r}_{2}}{{(S{{O}_{4}})}_{3}}\]

    D)
    \[N{{a}_{2}}S{{O}_{4}},\,\,S{{O}_{2}},\,C{{r}_{2}}{{(S{{O}_{4}})}_{3}}\]

    View Answer play_arrow
  • question_answer115)  Molecules of a noble gas do not possess vibrational energy because a noble gas

    A)
    is monoatomic

    B)
    is chemically inert

    C)
    has completely filled shells

    D)
    is diamagnetic

    View Answer play_arrow
  • question_answer116)  \[1\,\,d{{m}^{3}}\] solution containing \[{{10}^{-5}}\] moles each of \[C{{l}^{-}}\] ions and \[CrO_{4}^{2-}\] ions is treated with \[{{10}^{-4}}\] moles of silver nitrate. Which one of the following observations is made? \[[{{K}_{sp}}A{{g}_{2}}Cr{{O}_{4}}=4\times {{10}^{-12}}]\]

    A)
    Precipitation does not occur

    B)
    Silver chromate gets precipitated first

    C)
    Silver chloride gets precipitated first

    D)
    Both silver chromate and silver chloride start precipitating simultaneously

    View Answer play_arrow
  • question_answer117)  pH value of which one of the following is not equal to one?

    A)
    \[0.1\text{ }M\text{ }HN{{O}_{3}}\]

    B)
    \[0.05\text{ }M\text{ }{{H}_{2}}S{{O}_{4}}\]

    C)
    \[0.1\text{ }M\text{ }C{{H}_{3}}COOH\]

    D)
    \[50\,\,c{{m}^{3}}\]of \[0.4\,\,M\]\[HCl+50\,\,c{{m}^{3}}\]of \[0.2\,M\,NaOH\]

    View Answer play_arrow
  • question_answer118)    \[{{E}_{1}},\,{{E}_{2}},{{E}_{3}}\] are the emf values of the three galvanic cells respectively.      
    (i) \[Zn|Zn_{1M}^{2+}||Cu_{0.1M}^{2+}|Cu\]
    (ii) \[Zn|Zn_{1M}^{2+}||Cu_{1M}^{2+}|Cu\]
    (iii) \[Zn|Zn_{0.1\,M}^{2+}||Cu_{1M}^{2+}|Cu\]
          Which one of the following is true?  

    A)
    \[{{E}_{2}}>{{E}_{3}}>{{E}_{1}}\]              

    B)
    \[{{E}_{3}}>{{E}_{2}}>{{E}_{1}}\]

    C)
    \[{{E}_{1}}>{{E}_{2}}>{{E}_{3}}\]                              

    D)
    \[{{E}_{1}}>{{E}_{3}}>{{E}_{2}}\]

    View Answer play_arrow
  • question_answer119)    The IUPAC name of is  

    A)
      2-methyl-3-bromohexanal

    B)
      - 3-bromo-2-methylbutanal

    C)
      2-methyl-3-bromobutanal

    D)
      3-bromo-2-methylpentanal

    View Answer play_arrow
  • question_answer120)  Which one of the following forms propane nitrile as the major product?

    A)
    Ethyl bromide + alcoholic \[KCN\]

    B)
    Propyl bromide + alcoholic \[KCN\]

    C)
    Propyl bromide + alcoholic \[AgCN\]

    D)
    Ethyl bromide + alcoholic \[AgCN\]

    View Answer play_arrow
  • question_answer121)  If then \[{{A}^{2}}+xA+I=0\]  for \[(x,y)\] is

    A)
      \[(-4,1)\]                             

    B)
      \[(-1,3)\]

    C)
      \[(4,-1)\]                             

    D)
      \[(1,3)\]

    View Answer play_arrow
  • question_answer122)  The constant term of the polynomial is

    A)
    0

    B)
    2

    C)
    -1

    D)
    1

    View Answer play_arrow
  • question_answer123)  If \[\vec{a},\vec{b}\] and \[\vec{c}\] are non-zero coplanar vectors, then \[[2\vec{a}-\vec{b}3\vec{b}-\vec{c}4\vec{c}-\vec{a}]\] is

    A)
    \[25\]                                   

    B)
    \[0\]

    C)
    \[27\]                                   

    D)
    \[9\]

    View Answer play_arrow
  • question_answer124)  A space vector makes the angles \[{{150}^{o}}\] and \[{{60}^{o}}\] with the positive direction of x-and y-axes. The angle made by the vector with the positive direction z-axis is

    A)
    \[{{90}^{o}}\]                                    

    B)
    \[{{60}^{o}}\]

    C)
    \[{{180}^{o}}\]                                 

    D)
    \[{{120}^{o}}\]

    View Answer play_arrow
  • question_answer125)  If \[\vec{a},\vec{b}\]and \[\vec{c}\] are unit vectors, such that \[\vec{a}+\vec{b}+\vec{c}=\vec{0},\]then  \[3\vec{a}.\vec{b}+2\,\vec{b}.\vec{c}+\vec{c}.\vec{a}\]

    A)
    -1

    B)
    1

    C)
    -3

    D)
    3

    View Answer play_arrow
  • question_answer126)  If \[a>b>c,\] \[{{\sec }^{-1}}\frac{a+b}{a-b}=2{{\sin }^{-1}}x,\] then x is

    A)
    \[-\sqrt{\frac{b}{a+b}}\]                              

    B)
    \[\sqrt{\frac{b}{a+b}}\]

    C)
    \[-\sqrt{\frac{a}{a+b}}\]                              

    D)
    \[\sqrt{\frac{a}{a+b}}\]

    View Answer play_arrow
  • question_answer127)  If \[x\ne n\pi ,\] \[x\ne (2n+1)\frac{\pi }{2},\] \[n\in Z,\] then \[\frac{{{\sin }^{-1}}(\cos x)+{{\cos }^{-1}}(\sin x)}{{{\tan }^{-1}}(\cot x)+{{\cot }^{-1}}(\tan x)}\] is

    A)
    \[\frac{\pi }{2}\]                                              

    B)
    \[\frac{\pi }{6}\]

    C)
    \[\frac{\pi }{4}\]                                              

    D)
    \[\frac{\pi }{3}\]

    View Answer play_arrow
  • question_answer128)  The general solution \[1+{{\sin }^{2}}3\sin x.\cos x,\tan x\ne \frac{1}{2},\]is

    A)
    \[2n\pi +\frac{\pi }{4},n\in Z\]

    B)
    \[2n\pi -\frac{\pi }{4},n\in Z\]

    C)
    \[n\pi -\frac{\pi }{4},n\in Z\]

    D)
    \[n\pi +\frac{\pi }{4},n\in Z\]

    View Answer play_arrow
  • question_answer129)  The least positive integer n, for which \[\frac{{{(1+i)}^{n}}}{{{(1-i)}^{n-2}}}\] is positive, is

    A)
    3

    B)
    4

    C)
    1

    D)
    2

    View Answer play_arrow
  • question_answer130)  If \[x+iy={{(-1+i\sqrt{3})}^{2010}},\] then x is

    A)
    \[-{{2}^{2010}}\]                              

    B)
    \[{{2}^{2010}}\]

    C)
    \[1\]                                     

    D)
    \[-1\]

    View Answer play_arrow
  • question_answer131)  \[(\sin \theta +\cos \theta )\,(\tan \theta +cot\theta )\] is equal to

    A)
    \[\sin \theta \,\cos \theta \]

    B)
    \[1\]

    C)
    \[\sec \theta +\text{cosec}\theta \]

    D)
    \[\sec \theta \,\,\text{cosec}\theta \]

    View Answer play_arrow
  • question_answer132)  This sides of a triangle are \[6+2\sqrt{3},4\sqrt{3}\] and \[\sqrt{24}.\] the tangent of the smallest angle of the triangle is

    A)
    \[\frac{1}{\sqrt{3}}\]                                     

    B)
    \[\sqrt{2}-1\]

    C)
    \[\sqrt{3}\]                                        

    D)
    \[1\]

    View Answer play_arrow
  • question_answer133)  A simple graph contains 24 edges. Degree of each vertex is 3. The number of vertices is

    A)
    8

    B)
    12

    C)
    21

    D)
    16

    View Answer play_arrow
  • question_answer134)  \[\underset{x\to \infty }{\mathop{\lim }}\,\,n\,\sin \frac{2\pi }{3n}.\cos \frac{2\pi }{3n}\] is

    A)
    \[\frac{\pi }{6}\]                                              

    B)
    \[\frac{2\pi }{3}\]

    C)
    \[1\]                                     

    D)
    \[\frac{\pi }{3}\]

    View Answer play_arrow
  • question_answer135)  The function \[f(x)=[x],\]  where [x] denotes the greatest integer not greater than x, is

    A)
    continuous for all non-integral values of x

    B)
    continuous only at positive integral values of x

    C)
    continuous for all real values of x

    D)
    continuous only at rational values of x

    View Answer play_arrow
  • question_answer136)  The greatest value of x satisfying \[~21=385\](mod x) and \[587=167\] (mod x) is

    A)
    \[156\]                                 

    B)
    \[32\]

    C)
    \[28\]                                   

    D)
    \[56\]

    View Answer play_arrow
  • question_answer137)  The number \[({{49}^{2}}-4)({{49}^{3}}-49)\] is divisible by

    A)
    \[7!\]                                    

    B)
    \[9!\]

    C)
    \[6!\]                                    

    D)
    \[5!\]

    View Answer play_arrow
  • question_answer138)  The least positive integer x satisfying\[{{2}^{2010}}\equiv 3x\](mod 5) is

    A)
    3

    B)
    4

    C)
    1

    D)
    2

    View Answer play_arrow
  • question_answer139)  If A and B are two square matrices of the same order such that \[AB=B\]and \[BA=A,\] then \[{{A}^{2}}+{{B}^{2}}\]is always equal to

    A)
    \[I\]                                      

    B)
    \[A+B\]

    C)
    \[2AB\]                                

    D)
    \[2BA\]

    View Answer play_arrow
  • question_answer140)  If A is a \[3\times 3\] non-singular matrix and if \[|A|=3,\]then \[|{{(2A)}^{-1}}|\] is

    A)
    \[24\]                                   

    B)
    \[3\]

    C)
    \[\frac{1}{3}\]                                   

    D)
    \[\frac{1}{24}\]

    View Answer play_arrow
  • question_answer141)  If   \[a,-a,\] b   are   the   roots   of \[{{x}^{3}}-5{{x}^{2}}-x+5=0,\]  then b is a root of

    A)
    \[{{x}^{2}}+3x-20=0\]

    B)
    \[{{x}^{2}}-5x+10=0\]

    C)
    \[{{x}^{2}}-3x-10=0\]

    D)
      \[{{x}^{2}}+5x-30=0\]  

    View Answer play_arrow
  • question_answer142)  In the binomial expansion of \[{{(1+x)}^{15}},\] the coefficients of \[{{x}^{r}}\] and \[{{x}^{r+3}}\] are equal. Then, r is

    A)
    \[8\]                                     

    B)
    \[7\]

    C)
    \[4\]                                     

    D)
    \[6\]

    View Answer play_arrow
  • question_answer143)  The   nth   term   of   the   series \[1+3+7+13+21+.....\] is 9901. The value of n is

    A)
    \[100\]                                 

    B)
    \[90\]

    C)
    \[900\]                                 

    D)
    \[99\]

    View Answer play_arrow
  • question_answer144)  If \[\frac{1}{(3-5x)(2+3x)}=\frac{A}{3-5x}+\frac{B}{2+3x},\]  then \[A:B\] is

    A)
    \[2:3\]                                  

    B)
    \[5:3\]

    C)
    \[3:5\]                                  

    D)
    \[3:2\]

    View Answer play_arrow
  • question_answer145)  If \[\hat{i},\hat{j},\hat{k}\] are unit vectors along the positive direction of x, y and z-axes, then a false statement in the following is

    A)
    \[\Sigma \hat{i}\times (\hat{j}+\hat{k})=\vec{0}\]

    B)
    \[\Sigma \hat{i}\times (\hat{j}\times \hat{k})=\vec{0}\]

    C)
    \[\Sigma \,\hat{i}.(\hat{j}\times \hat{k})=\vec{0}\]

    D)
    \[\Sigma \,\hat{i}.(\hat{j}+\hat{k})=\vec{0}\]

    View Answer play_arrow
  • question_answer146)  In P(X), the power set of a non-empty set X, an binary operation * is defined by\[A\,*\,B=A\cup B,\forall A,B\in P(x)\] under *, a true statement is

    A)
    identity law is not satisfied

    B)
    inverse law is not satisfied

    C)
    commutative law is not satisfied

    D)
    associative law is not satisfied

    View Answer play_arrow
  • question_answer147)  The inverse of 2010 in the group \[{{Q}^{+}}\] of all positive rational under the binary operation * defined by \[a*b=\frac{ab}{2010},\forall \,a,b\in {{Q}^{+}},\] is

    A)
    \[2009\]                              

    B)
    \[2011\]

    C)
    \[1\]                                     

    D)
    \[2010\]

    View Answer play_arrow
  • question_answer148)  If the three function \[f(x),g(x)\] and \[h(x)\] are such that \[h(x)=f(x).g(x)\] and \[f'(x).g'(x)=c\] where c is constant, then

    A)
    \[h'=(x).h''(x)\]

    B)
    \[\frac{h(x)}{h''(x)}\]

    C)
    \[\frac{h''(x)}{h(x)}\]

    D)
    \[\frac{h(x)}{h'(x)}\]

    View Answer play_arrow
  • question_answer149)  The derivative of \[{{e}^{ax}}\,\,\cos \,bx\]with respect x is \[r{{e}^{ax}}\,\,\cos \,bx\,\,{{\tan }^{-1}}\frac{b}{a}.\] When \[a>0,b>0,\]then value of r, is

    A)
    \[\sqrt{{{a}^{2}}+{{b}^{2}}}\]

    B)
    \[\frac{1}{\sqrt{ab}}\]

    C)
    \[ab\]

    D)
    \[a+b\]

    View Answer play_arrow
  • question_answer150)  The chord of the circle \[{{x}^{2}}+{{y}^{2}}-4x=0\] which is bisected at \[(1,0)\] is perpendicular to the line

    A)
    \[y=x\]

    B)
    \[x+y=0\]

    C)
    \[x=1\]

    D)
    \[y=1\]        

    View Answer play_arrow
  • question_answer151)  In \[\Delta ABC,\]if \[a=2,b={{\tan }^{-1}}\frac{1}{2}\] and \[C={{\tan }^{-1}}\frac{1}{3},\] then (A, b) equals

    A)
    \[\frac{3\pi }{4},\frac{2}{\sqrt{5}}\]

    B)
    \[\frac{\pi }{4},\frac{2\sqrt{2}}{\sqrt{5}}\]

    C)
    \[\frac{3\pi }{4},\frac{2\sqrt{2}}{\sqrt{5}}\]

    D)
    \[\frac{\pi }{4},\frac{2}{\sqrt{5}}\]

    View Answer play_arrow
  • question_answer152)  The straight line \[2x+3y-k=0,\] \[k>0\] cuts the x and y-axes at A and B. The area of \[\Delta OAB,\], where 0 is the origin, is \[12\text{ }sq\]unit.    The equation of the circle having AB as diameter is

    A)
    \[{{x}^{2}}+{{y}^{2}}-6x-4y=0\]

    B)
    \[{{x}^{2}}+{{y}^{2}}+4x-6y=0\]

    C)
    \[{{x}^{2}}+{{y}^{2}}-6x+4y=0\]

    D)
    \[{{x}^{2}}+{{y}^{2}}-4x-6y=0\]

    View Answer play_arrow
  • question_answer153)  Let \[P(x,y)\] be the mid point of the line joining \[(1,0)\] to a point on the curve Then, locus of P is symmetrical about

    A)
      \[y-axis\]

    B)
      \[x-axis\]

    C)
      \[x=1\]

    D)
      \[y=1\]

    View Answer play_arrow
  • question_answer154)  The function \[f(x)=|x-2|+x\] is

    A)
    differentiable at both \[x=2\] and \[x=0\]

    B)
    differentiable at \[x=2\] but not at \[x=0\]

    C)
    continuous at \[x=2\]but not at \[x=0\]

    D)
    continuous at both \[x=2\] and \[x=0\]

    View Answer play_arrow
  • question_answer155)  Let R be an equivalence relation defined on a set containing 6 elements. The minimum number or ordered pairs that R should contain is

    A)
    \[12\]                                   

    B)
    \[6\]    

    C)
    \[64\]                                   

    D)
    \[36\]

    View Answer play_arrow
  • question_answer156)  The line joining \[A(2,-7)\] and \[B(6,5)\] is divided into 4 equal parts by the points P, Q and R such that\[AQ=RP=QB\]. The mid-point of PR is

    A)
    \[(4,12)\]                            

    B)
    \[(-8,1)\]

    C)
    \[(4,-1)\]                             

    D)
    \[(8,-2)\]

    View Answer play_arrow
  • question_answer157)  Let \[P\equiv (-1,0),\] \[Q\equiv (0,0)\] and \[R=(3,3)\] be three points. The equation of the bisector of the angle PQR is

    A)
    \[x-\sqrt{3}y=0\]    

    B)
    \[\sqrt{3}x-y=0\]

    C)
    \[x+\sqrt{3}y=0\]    

    D)
    \[\sqrt{3}x+y=0\]

    View Answer play_arrow
  • question_answer158)  If m is the slope of one of the lines represented by \[a{{x}^{2}}+2hxy+b{{y}^{2}}=0,\]then \[{{(h+bm)}^{2}}\] is equal to

    A)
    \[{{(a+b)}^{2}}\]

    B)
    \[{{(a-b)}^{2}}\]

    C)
    \[{{h}^{2}}+ab\]

    D)
    \[{{h}^{2}}-ab\]

    View Answer play_arrow
  • question_answer159)  \[\cos {{12}^{o}}\,\cot {{102}^{o}}+\cot {{102}^{o}}\,\cot {{66}^{o}}\]\[+\cot {{66}^{o}}\,\cot {{12}^{o}}\] is

    A)
    \[-2\]                                    

    B)
    \[1\]

    C)
    \[-1\]                                    

    D)
    \[2\]

    View Answer play_arrow
  • question_answer160)  A wire of length \[20\text{ }cm\]is bent in the form of a sector of a circle. The maximum area that can be enclosed by the wire is

    A)
    \[20\text{ }sq\text{ }cm\]       

    B)
    \[25\text{ }sq\text{ }cm\]

    C)
    \[10\,\,sq\,\,cm\]       

    D)
    \[30\,\,sq\,\,cm\]

    View Answer play_arrow
  • question_answer161)  Two circles centred at \[(2,3)\] and \[(5,6)\] intersect each other. If the radii are equal, the equation of the common chord is

    A)
    \[x+y+1=0\]

    B)
    \[x-y+1=0\]

    C)
    \[x+y-8=0\]

    D)
    \[x-y-8=0\]

    View Answer play_arrow
  • question_answer162)  Equation of the circle centred at \[(4,3)\] touching the circle \[{{x}^{2}}+{{y}^{2}}=1\]externally, is

    A)
    \[{{x}^{2}}+{{y}^{2}}-8x-6y+9=0\]

    B)
    \[{{x}^{2}}+{{y}^{2}}+8x+6y+9=0\]

    C)
    \[{{x}^{2}}+{{y}^{2}}+8x-6y+9=0\]

    D)
    \[{{x}^{2}}+{{y}^{2}}-8x6y+9=0\]

    View Answer play_arrow
  • question_answer163)  The points \[(1,0),\] \[(0,1),\] \[(0,0)\]  and \[(2k,3k),k\ne 0\]  are concyclic, if k is

    A)
    \[\frac{1}{5}\]                                   

    B)
    \[-\frac{1}{5}\]

    C)
    \[-\frac{5}{13}\]                                               

    D)
    \[\frac{5}{13}\]

    View Answer play_arrow
  • question_answer164)  The locus of the point of intersection of the tangents drawn at the ends of a focal chord of the parabola \[{{x}^{2}}=-\text{ }8y\] is

    A)
    \[x=2\]                                

    B)
    \[x=-2\]

    C)
    \[y=2\]                                

    D)
    \[y=-2\]

    View Answer play_arrow
  • question_answer165)  The condition for the line \[y=mx+c\]to be a normal to the parabola \[{{y}^{2}}=4ax\] is

    A)
    \[c=-2am-a{{m}^{3}}\]

    B)
    \[c=-\frac{a}{m}\]

    C)
    \[c=\frac{a}{m}\]

    D)
    \[c=2am+a{{m}^{3}}\]

    View Answer play_arrow
  • question_answer166)  The eccentric angle of the point \[(2,\sqrt{3})\]  lying on \[\frac{{{x}^{2}}}{16}+\frac{{{y}^{2}}}{4}=1\] is

    A)
    \[\frac{\pi }{4}\]                                              

    B)
    \[\frac{\pi }{2}\]

    C)
    \[\frac{\pi }{3}\]                                              

    D)
    \[\frac{\pi }{6}\]

    View Answer play_arrow
  • question_answer167)  The distance of the focus of \[{{x}^{2}}-{{y}^{2}}=4,\]form the directrix which is nearer to it, is

    A)
    \[4\sqrt{2}\]                                      

    B)
    \[8\sqrt{2}\]

    C)
    \[2\sqrt{2}\]                                      

    D)
    \[\sqrt{2}\]

    View Answer play_arrow
  • question_answer168)  If \[\int{f(x)\,\sin x.\cos x\,dx}\]\[=\frac{1}{2({{b}^{2}}-{{a}^{2}})}\log f(x)+c,\]where c is the constant of integration, then \[f(x)\] is

    A)
    \[\frac{2}{ab\,\,\cos \,\,2x}\]

    B)
    \[\frac{2}{({{b}^{2}}-{{a}^{2}})\,\cos \,2x}\]

    C)
    \[\frac{2}{ab\,\sin \,2x}\]

    D)
    \[\frac{2}{({{b}^{2}}-{{a}^{2}})\,\sin \,2x}\]

    View Answer play_arrow
  • question_answer169)  If  \[\int{\frac{\sqrt{x}}{x(x+1)}}\,dx=k\,{{\tan }^{-1}}m,\]then \[(k,m)\] is

    A)
    \[(2,x)\]                              

    B)
    \[(1,x)\]

    C)
    \[(1,\sqrt{x})\]                 

    D)
    \[(2,\sqrt{x})\]

    View Answer play_arrow
  • question_answer170)  \[\int_{0}^{\pi /4}{\frac{\sin \,x+\cos x}{3+\sin 2x}}dx\] is

    A)
    \[\frac{1}{4}\log 3\]

    B)
    \[\log 3\]

    C)
    \[\frac{1}{2\log \,3}\]

    D)
    \[2\log 3\]

    View Answer play_arrow
  • question_answer171)  \[\int_{0}^{1}{x{{(1-x)}^{3/2}}}dx\] is

    A)
    \[-\frac{2}{35}\]                               

    B)
    \[\frac{4}{35}\]

    C)
      \[\frac{24}{35}\]                                                

    D)
    \[-\frac{8}{35}\]

    View Answer play_arrow
  • question_answer172)  The area bounded by the curve \[y=\left\{ \begin{matrix}    {{x}^{2}}, & x<0  \\    x, & x\ge 0  \\ \end{matrix} \right.\] and the line \[y=4,\] is

    A)
    \[\frac{32}{3}\]                                

    B)
    \[\frac{8}{3}\]

    C)
    \[\frac{40}{3}\]                                

    D)
    \[\frac{16}{3}\]

    View Answer play_arrow
  • question_answer173)  The order and degree of the differential equation   \[y=\frac{dp}{dx}x=\sqrt{{{a}^{2}}{{p}^{2}}+{{b}^{2}}},\]where  \[p=\frac{dy}{dx}\] (here a and b are arbitrary constants) respectively are

    A)
    \[2,2\]                                  

    B)
    \[1,1\]

    C)
    \[1,2\]                                  

    D)
    \[2,1\]

    View Answer play_arrow
  • question_answer174)  The general solution of the differential equation \[2x\frac{dy}{dx}-y=3\] is a family of

    A)
    hyperbolas      

    B)
    parabolas

    C)
    straight lines   

    D)
    circles

    View Answer play_arrow
  • question_answer175)  If \[x=a\,{{\cos }^{3}}\theta \]and \[y=a\,{{\sin }^{3}}\,\theta ,\] then \[\frac{dy}{dx}\] is

    A)
    \[\sqrt[3]{\frac{y}{x}}\]                                

    B)
    \[\sqrt[3]{\frac{x}{y}}\]

    C)
    \[-\sqrt[3]{\frac{x}{y}}\]                              

    D)
    \[-\sqrt[3]{\frac{y}{x}}\]

    View Answer play_arrow
  • question_answer176)  If \[y={{\tan }^{-1}}\sqrt{{{x}^{2}}-1},\]  then the ratio \[\frac{{{d}^{2}}y}{d{{x}^{2}}}:\frac{dy}{dx}\] is

    A)
    \[\frac{x({{x}^{2}}-1)}{1+2{{x}^{2}}}\]

    B)
    \[\frac{1-2{{x}^{2}}}{x({{x}^{2}}-1)}\]

    C)
      \[\frac{1+2{{x}^{2}}}{x({{x}^{2}}+1)}\]  

    D)
    \[\frac{x({{x}^{2}}+1)}{1-2{{x}^{2}}}\]

    View Answer play_arrow
  • question_answer177)  P is the point of contact of the tangent form the origin to the curve\[y=lo{{g}_{e}}\text{ }x\]. The length of the perpendicular drawn form the origin to the normal at P is

    A)
    \[\frac{1}{2e}\]                                

    B)
    \[\frac{1}{e}\]

    C)
    \[2\sqrt{{{e}^{2}}+1}\]                 

    D)
    \[\sqrt{{{e}^{2}}+1}\]

    View Answer play_arrow
  • question_answer178)  For the curve \[4{{x}^{5}}=5{{y}^{4}},\] the ratio of the    cube of the subtangent at a point on the curve the square of the subnormal at the same point is

    A)
    \[\frac{{{4}^{4}}}{5}\]                                    

    B)
    \[\frac{{{5}^{4}}}{4}\]

    C)
    \[\frac{{{4}^{4}}}{{{5}^{4}}}\]                                     

    D)
    \[{{\left( \frac{5}{4} \right)}^{4}}\]

    View Answer play_arrow
  • question_answer179)  The set of real values of x for which \[f(x)=\frac{x}{\log \,x}\]is increasing, is

    A)
    \[\{x:x\ge e\}\]

    B)
    empty

    C)
    \[\{x:x<e\}\]

    D)
    \[\{1\}\]

    View Answer play_arrow

Study Package

   


You need to login to perform this action.
You will be redirected in 3 sec spinner